Difference between revisions of "1981 USAMO Problems/Problem 3"
(Created page with "==Problem== Show that for any triangle, <math>\frac{3\sqrt{3}}{2}\ge \sin(3A) + \sin(3B) + \sin (3C) \ge -2</math>. When does the equality hold? ==Solution== {{solution}} ==S...") |
(LaTeX overhaul) |
||
(4 intermediate revisions by 4 users not shown) | |||
Line 5: | Line 5: | ||
==Solution== | ==Solution== | ||
− | {{ | + | Given three angles that add to <math>180^\circ</math>, one can construct a triangle from them. However, its angles must all be nonnegative; thus the constraints on the angles of a triangle are <math>0^\circ\le A,B,C\le180^\circ</math> and <math>A+B+C=180^\circ</math>. |
+ | |||
+ | In fact, at this point, we only care about <math>3A, 3B,</math> and <math>3C</math>. Let us call them <math>x, y,</math> and <math>z</math>. We have: | ||
+ | |||
+ | <cmath>0 \le x,y,z \le 540^\circ, x+y+z = 540^\circ</cmath> | ||
+ | |||
+ | and we must prove that | ||
+ | |||
+ | <cmath>\frac{3\sqrt{3}}{2}\ge \sin(x) + \sin(y) + \sin (z) \ge -2</cmath> | ||
+ | |||
+ | Without loss of generality, assume <math>x \le y \le z</math>. It follows that <math>x \le 180^\circ</math> and <math>z \ge 180^\circ</math> (otherwise, <math>x+y+z</math> would be strictly greater than <math>180^\circ</math> or strictly less than <math>180^\circ</math>, respectively). | ||
+ | |||
+ | Since <math>\sin(u)</math> is nonnegative over the interval <math>[0^\circ, 180^\circ]</math> and <math>-1 \le \sin (u) \le 1</math> for all real <math>u</math>, we can immediately use the <math>x \le 180^\circ</math> result to show that | ||
+ | |||
+ | <cmath>\sin( x) + \sin (y) + \sin (z) \le 0 + -1 + -1 = -2</cmath> | ||
+ | |||
+ | This proves the lower bound; equality occurs when <math>\sin(x) = 0</math> and <math>\sin (y) = \sin (z) = -1</math>, and this is reachable only when <math>y = z = 270^\circ</math> and <math>x = 0^\circ</math>, which translates into <math>A = 0^\circ</math> and <math>B = C = 90^\circ</math>. | ||
+ | |||
+ | Now for the upper bound. It is true that <math>\sin(u)</math> is non-positive over the interval <math>[180^\circ, 360^\circ]</math>. Also, <math>\frac{3\sqrt{3}}{2}>2</math> (by a simple squaring argument). Then <math>z \ge 180^\circ</math>. If <math>z \le 360^\circ</math>, then <math>\sin (z) \le 0</math>, and then | ||
+ | <cmath>\sin (x) + \sin (y) + \sin (z) \le 1 + 1 + 0 = 2 < \frac{3\sqrt{3}}{2}</cmath> | ||
+ | Therefore, we need to handle the case where <math>z \ge 360^\circ</math>. <math>\sin(u + 360^\circ) = \sin(u)</math>, so we can subtract <math>360^\circ</math> from <math>z</math> due to periodicity. Then the constraints become <math>0^\circ \le x,y,z \le 180^\circ</math> and <math>x+y+z = 180^\circ</math>. | ||
+ | |||
+ | Let us suppose that <math>(x,y,z)</math> is the selection of <math>x,y,z</math> given the above constraints with the largest value of <math>\sin (x) + \sin (y) + \sin (z)</math>. We shall see that, given these constraints, this value increases when the distance between two of these variables decreases, and it is maximized when <math>x=y=z</math>. | ||
+ | |||
+ | <math>\textbf{Lemma:}</math> If <math>f(x)</math> is well-behaved and <math>f'(x)</math> is strictly decreasing over an open interval <math>(a, b)</math>, then, for any <math>x,y</math> selected from that interval with <math>x \ne y</math>, then <math>2f\left(\frac{x + y}{2}\right) > f(x) + f(y).</math> | ||
+ | |||
+ | <math>\textbf{Proof of lemma:}</math> It is true that, for any well-behaved function <math>f(x)</math>, <math>f(x+d) = f(x) + \int_{x}^{x+d} f'(u)\,du</math>, and likewise, <math>f(x-d) = f(x) - \int_{x-d}^x f'(u)\,du</math>. Without loss of generality, suppose <math>x < y</math>; let <math>d = \frac{1}{2}(y-x)</math>. Since <math>f'(u)</math> is given to be strictly decreasing over the relevant interval, we have <math>f'(u) > f'(x+d)</math> for all <math>u\in(x, x+d)</math>; therefore | ||
+ | <cmath>f(x+d) = f(x) + \int_x^{x+d} f'(u)\,du > f(x) + \int_x^{x+d} f'(x+d)\,du = f(x) + d\cdot f'(x+d)</cmath> | ||
+ | Likewise, <math>f'(u) < f'(y-d)</math> for all <math>u\in(y-d, y)</math>; therefore | ||
+ | <cmath>f(y-d) = f(y) - \int_{y-d}^y f'(u)\,du > f(y) - \int_{y-d}^y f'(y-d)\,du = f(y) - d\cdot f'(y-d)</cmath> | ||
+ | Therefore, adding our inequalities together, | ||
+ | <cmath>f(x+d) + f(y-d) > f(x) + d\cdot f'(x+d) + f(y) - d\cdot f'(y-d)</cmath> | ||
+ | But <math>x+d = y-d = \frac{x+y}{2}</math>. In that case, the <math>d\cdot f'(x+d)</math> and <math>-d\cdot f'(y-d)</math> terms cancel, and we get our desired result: <math>2 f\left(\frac{x+y}{2}\right) > f(x) + f(y)</math>. | ||
+ | |||
+ | Since <math>\sin(u)</math> is well-behaved, and <math>\sin'(u) = \cos(u)</math>, which is strictly decreasing over the open interval <math>(0^\circ, 180^\circ)</math>, then we can apply our lemma. Suppose for the sake of contradiction that <math>x,y,z</math> are not all the same value, and this produces a maximum. Then, without loss of generality, suppose <math>x \ne y</math>. Let <math>a = \frac{x+y}{2}</math>. Then the selection <math>(a,a,z)</math> also satisfies the constraints <math>0^\circ \le a,a,z \le 180^\circ</math> and <math>a+a+z \le 180^\circ</math>. Furthermore, by the above lemma, <math>2\cdot\sin (a) > \sin (x) + \sin (y)</math>, so this new selection <math>(a,a,z)</math> has a larger corresponding value <math>\sin (a) + \sin (a) + \sin (z)</math> than the selection <math>(x,y,z)</math>. This contradicts our original assumption that <math>(x,y,z)</math> was chosen to have the maximum. Therefore, <math>x=y=z</math>. | ||
+ | |||
+ | Then, since <math>x+y+z = 180^\circ</math>, we conclude <math>x=y=z=60^\circ</math>. Then | ||
+ | <cmath>\sin (x) + \sin (y) + \sin (z) = 3 \cdot \sin (60^\circ) = 3 \cdot \frac{\sqrt{3}}{2}</cmath> | ||
+ | So <math>\frac{\sqrt{3}}{2}</math> is indeed the maximum value of <math>\sin(x) + \sin (y) + \sin (z)</math>, proving the bound. Translating back into the previous problem, the maximum occurs when <math>x=y=60^\circ</math> and <math>z=420^\circ</math>; translating into the original problem, we have proven the upper bound, and equality with the upper bound occurs when <math>A=B=20^\circ</math> and <math>C=140^\circ</math>. | ||
+ | |||
+ | ~ <math>\LaTeX</math> by [https://artofproblemsolving.com/wiki/index.php/User:Eevee9406 eevee9046] | ||
==See Also== | ==See Also== | ||
{{USAMO box|year=1981|num-b=2|num-a=4}} | {{USAMO box|year=1981|num-b=2|num-a=4}} | ||
+ | {{MAA Notice}} | ||
[[Category:Olympiad Trigonometry Problems]] | [[Category:Olympiad Trigonometry Problems]] |
Latest revision as of 00:05, 13 March 2025
Problem
Show that for any triangle, .
When does the equality hold?
Solution
Given three angles that add to , one can construct a triangle from them. However, its angles must all be nonnegative; thus the constraints on the angles of a triangle are
and
.
In fact, at this point, we only care about and
. Let us call them
and
. We have:
and we must prove that
Without loss of generality, assume . It follows that
and
(otherwise,
would be strictly greater than
or strictly less than
, respectively).
Since is nonnegative over the interval
and
for all real
, we can immediately use the
result to show that
This proves the lower bound; equality occurs when and
, and this is reachable only when
and
, which translates into
and
.
Now for the upper bound. It is true that is non-positive over the interval
. Also,
(by a simple squaring argument). Then
. If
, then
, and then
Therefore, we need to handle the case where
.
, so we can subtract
from
due to periodicity. Then the constraints become
and
.
Let us suppose that is the selection of
given the above constraints with the largest value of
. We shall see that, given these constraints, this value increases when the distance between two of these variables decreases, and it is maximized when
.
If
is well-behaved and
is strictly decreasing over an open interval
, then, for any
selected from that interval with
, then
It is true that, for any well-behaved function
,
, and likewise,
. Without loss of generality, suppose
; let
. Since
is given to be strictly decreasing over the relevant interval, we have
for all
; therefore
Likewise,
for all
; therefore
Therefore, adding our inequalities together,
But
. In that case, the
and
terms cancel, and we get our desired result:
.
Since is well-behaved, and
, which is strictly decreasing over the open interval
, then we can apply our lemma. Suppose for the sake of contradiction that
are not all the same value, and this produces a maximum. Then, without loss of generality, suppose
. Let
. Then the selection
also satisfies the constraints
and
. Furthermore, by the above lemma,
, so this new selection
has a larger corresponding value
than the selection
. This contradicts our original assumption that
was chosen to have the maximum. Therefore,
.
Then, since , we conclude
. Then
So
is indeed the maximum value of
, proving the bound. Translating back into the previous problem, the maximum occurs when
and
; translating into the original problem, we have proven the upper bound, and equality with the upper bound occurs when
and
.
~ by eevee9046
See Also
1981 USAMO (Problems • Resources) | ||
Preceded by Problem 2 |
Followed by Problem 4 | |
1 • 2 • 3 • 4 • 5 | ||
All USAMO Problems and Solutions |
These problems are copyrighted © by the Mathematical Association of America, as part of the American Mathematics Competitions.